LSAT and Law School Admissions Forum

Get expert LSAT preparation and law school admissions advice from PowerScore Test Preparation.

User avatar
 Dave Killoran
PowerScore Staff
  • PowerScore Staff
  • Posts: 5852
  • Joined: Mar 25, 2011
|
#87759
Complete Question Explanation
(The complete setup for this game can be found here: lsat/viewtopic.php?f=155&t=1451)

The correct answer choice is (B)

If G auditions on both Wednesday and Saturday, then in order to meet the second rule (and comply with the first rule), R must audition on Saturday also (with the R2 audition):

G2-Q11-d1.png

At this point, several different scenarios could occur. For example, R1 could audition on Thursday (thus satisfying the third rule; O would audition Wednesday and Friday). Or, O could audition on Thursday and Saturday, in order to satisfy the third rule; R1 would audition on Friday).

As stated in the paragraph above, O could audition on Wednesday and Friday, and thus answer choice (B) could be true and is correct.

The following details the problems with the remaining four answer choices:

Answer choice (A): This scenario would cause a violation of the third rule.

Answer choice (C): This scenario would leave Friday without an actor audition (or, if R1 auditioned on Friday, cause a violation of the third rule).

Answer choice (D): This scenario is impossible because R cannot audition on Wednesday.

Answer choice (E): This scenario would cause an immediate violation of the second rule (which initially had caused R2 to audition on Saturday).
You do not have the required permissions to view the files attached to this post.

Get the most out of your LSAT Prep Plus subscription.

Analyze and track your performance with our Testing and Analytics Package.